So what is the correct answer?

Forum organization and occasional community-building.
Forum rules
Questions about Ren'Py should go in the Ren'Py Questions and Announcements forum.

Read the question and tell me what answer you think is right

A: 1/4
1
6%
B: 1/3
2
12%
C: 1/2
13
76%
D: 2/3
1
6%
 
Total votes: 17

Message
Author
Counter Arts
Miko-Class Veteran
Posts: 649
Joined: Fri Dec 16, 2005 5:21 pm
Completed: Fading Hearts, Infinite Game Works
Projects: Don't Save the World
Organization: Sakura River
Location: Canada
Contact:

#31 Post by Counter Arts »

Beta wrote:
Counter Arts wrote:Assume Beta's theory is true. Then the possiblity for the outcomes are...

( 1 )

HHX = 1/6
HTH = 1/6
HTT = 1/6
THT = 1/6
THH = 1/6
TTX = 1/6

I'm sure we all can agree that...

( 2 )

HH = 1/4
TH = 1/4
HT = 1/4
TT = 1/4

We can derive from ( 1 ) the following

HHX = 1/6 = HH
HTX = 2/6 = HT
THX = 2/6 = TH
TTX = 1/6 = TT

This contradicts ( 2 ) therefore Beta's theory is inconsistent.
and your proof conflicts with the original problem. HTX and THX cannot occur because no team had won two games at that point, and the series only ends AFTER one team accumulates two victories. no sooner, no later. my solution is still valid at this point unless someone can point out the explicit wording in the original problem that contradicts this.

while you're at it, map out all the possible "routes" that a best two-out-of-three match can go when it ends immediately after one side wins two games.
Alright, I'll be wordy then.

Assume Beta's theory is true. Then the possiblity for the outcomes are...

( 1 )

HHX = 1/6
HTH = 1/6
HTT = 1/6
THT = 1/6
THH = 1/6
TTX = 1/6

I'm sure we all can agree that...

( 2 )

HH = 1/4
TH = 1/4
HT = 1/4
TT = 1/4

We can derive from ( 1 ) the following

HHX = 1/6 = HH
HTH or HTT = 2/6 = HT
THH or THT = 2/6 = TH

TTX = 1/6 = TT

This contradicts ( 2 ) therefore Beta's theory is inconsistent.

Can you prove me wrong by proving that the possiblity of HTH or HTT happening is not equal to the probablity that the first two games are HT?

herenvardo
Veteran
Posts: 359
Joined: Sat Feb 25, 2006 11:09 am
Location: Sant Cugat del Vallès (Barcelona, Spain)
Contact:

simplest argumentation

#32 Post by herenvardo »

This is what I find as the simplest, easiest to understand explanation about what it's 1/2
After the first game/flip, let's call A the option (head/tails or the team) that won it, and B the other option. Then, for the second game, chances are:
50% of A winning
50% of B winning
The case where A wins the second match is the case where the match ends after just 2 games, and the other one is the case where third game is required: so, there is a 50% chance.
Said in other words, the chance is that one for the second game to have the same winner that the first one, which is 1/2 (because there can be one winner from out of two teams).
Beta: find the right result by yourself, and if then you still disagree from us, let me know why. If you think for a while, you'll notice that the outcome of the third game is not relevant for the problem, you only care if there is or not a third game, and this can be inferred directly from the first two game's results. So, analyze only the possible results for these two first games and, from there, take the chance a third match will be required.

Anyhow, to put more discrepance on the discussion, I'll give some arguments in favor to another option.
:arrow: forget the coin flip and go back to the sport event. Assuming most neutral conditions, as playing in a neutral field and weather not affecting results, then it's most probable that the best team wins each game. So, after team A wins the first game, there is an important chance that A is better than B, so its chances to win the second game are higher than 50/50. With such an argumentation, the answer should be somewhat higher than 1/2, and the only option that fulfills that is 2/3. I have had many teachers that, provided an argumentation like the one above, would accept as good that option (but it will be taken as wrong if you mark it without any explanation).
Ok, it's not goog at all, but also isn't bad at all :P 8)
I have failed to meet my deadlines so many times I'm not announcing my projects anymore. Whatever I'm working on, it'll be released when it is ready :P

Beta
Regular
Posts: 66
Joined: Thu Apr 28, 2005 5:53 pm
Location: not where I was five seconds ago
Contact:

#33 Post by Beta »

well, we now have proof that all four answers can be backed up with an explanation (even if it is highly improbable). (PyTom is in fact partly responsible for answer A being shown possible.)

for Answer A to be even remotely correct, we'd have to assume there's an error with the question's wording. if the question was, in reality, to find the probability of a team winning THREE games in a set of three games, then this is the only answer possible, since the probability of one team winning ALL THREE GAMES is 1 in four, as PyTom (and a few others) has inadvertantly shown.
PyTom (others have shown this as well...) wrote:...Let's consider a sport which has a best-of-three championship series, but which, for whatever reason, demands that they play all three games, even if the outcome is preordained. We'd get the following possible outcomes:

HHH *
HHT *
HTH
HTT
THH
THT
TTH *
TTT *

The lines I've starred are the ones in which the games are won in two flips. All of the lines have the same probability, so the chance of someone winning in 2 games is 4/8 == 1/2.
note how 2 of the eight outcomes (which I have marked in green) is the same team winning all three times. (Gamma brought this to my attention.)

answer B, as I have shown and argued for a while about, assumes that the series ends after one team wins two games (contradictory to the explanation of answer A above). this results in only two of six possible ways that it could end after the second game with the third game not played at all.

for answer C, follow a similar logic but discard the probability calculation on the third game altogether. (this is how Zeta and most of his math class arrived at that solution...)

answer D has the weirdest (but still followable) logic behind it, so I'll just quote the person who explained it...
herenvardo wrote:forget the coin flip and go back to the sport event. Assuming most neutral conditions, as playing in a neutral field and weather not affecting results, then it's most probable that the best team wins each game. So, after team A wins the first game, there is an important chance that A is better than B, so its chances to win the second game are higher than 50/50. With such an argumentation, the answer should be somewhat higher than 1/2, and the only option that fulfills that is 2/3. I have had many teachers that, provided an argumentation like the one above, would accept as good that option (but it will be taken as wrong if you mark it without any explanation).
this leaves me to wonder if the people who wrote that question still have their jobs.

Counter Arts
Miko-Class Veteran
Posts: 649
Joined: Fri Dec 16, 2005 5:21 pm
Completed: Fading Hearts, Infinite Game Works
Projects: Don't Save the World
Organization: Sakura River
Location: Canada
Contact:

#34 Post by Counter Arts »

You still haven't proven me wrong by proving that the possiblity of HTH or HTT happening is not equal to the probablity that the first two games are HT? Just a reminder.

RedSlash
Veteran
Posts: 351
Joined: Sun Oct 31, 2004 12:48 am
Location: Canada
Contact:

#35 Post by RedSlash »

I don't really see the confusion between the wording. The question asks whats the probability of the game ending in 2 matches. Here's how I take it. There are only two cases.

1. Game ends in 2 matches. (Coin flips either HH or TT)
2. Game doesn't end in two matches. (Coin flips either HT or TH)

50%. The 3rd coin flip is irrelavent to whether the game ends in 2 matches or not, which is exactly what the question asks.

I think PyTom was kinda rubbing it in when he wrote that CGI script, however, it does answer the question as specified.

herenvardo
Veteran
Posts: 359
Joined: Sat Feb 25, 2006 11:09 am
Location: Sant Cugat del Vallès (Barcelona, Spain)
Contact:

#36 Post by herenvardo »

Beta wrote:well, we now have proof that all four answers can be backed up with an explanation (even if it is highly improbable). (PyTom is in fact partly responsible for answer A being shown possible.)

for Answer A to be even remotely correct, we'd have to assume there's an error with the question's wording. if the question was, in reality, to find the probability of a team winning THREE games in a set of three games, then this is the only answer possible, since the probability of one team winning ALL THREE GAMES is 1 in four, as PyTom (and a few others) has inadvertantly shown.
PyTom (others have shown this as well...) wrote:...Let's consider a sport which has a best-of-three championship series, but which, for whatever reason, demands that they play all three games, even if the outcome is preordained. We'd get the following possible outcomes:

HHH *
HHT *
HTH
HTT
THH
THT
TTH *
TTT *

The lines I've starred are the ones in which the games are won in two flips. All of the lines have the same probability, so the chance of someone winning in 2 games is 4/8 == 1/2.
note how 2 of the eight outcomes (which I have marked in green) is the same team winning all three times. (Gamma brought this to my attention.)

answer B, as I have shown and argued for a while about, assumes that the series ends after one team wins two games (contradictory to the explanation of answer A above). this results in only two of six possible ways that it could end after the second game with the third game not played at all.

for answer C, follow a similar logic but discard the probability calculation on the third game altogether. (this is how Zeta and most of his math class arrived at that solution...)

answer D has the weirdest (but still followable) logic behind it, so I'll just quote the person who explained it...
herenvardo wrote:forget the coin flip and go back to the sport event. Assuming most neutral conditions, as playing in a neutral field and weather not affecting results, then it's most probable that the best team wins each game. So, after team A wins the first game, there is an important chance that A is better than B, so its chances to win the second game are higher than 50/50. With such an argumentation, the answer should be somewhat higher than 1/2, and the only option that fulfills that is 2/3. I have had many teachers that, provided an argumentation like the one above, would accept as good that option (but it will be taken as wrong if you mark it without any explanation).
this leaves me to wonder if the people who wrote that question still have their jobs.
Well, you're right: all four answers can be argued for; but that doesn't make them right. In a pure math context, the only one that would be right is 1/2, and there have been a lot of different explanations, most of them unbreakable, about that one. About your arguing for B, I've found where does it fall, and I'll try to clearly explain it. You make a case-breakdown, like that:
HH
HTH
HTT
THH
THT
TT
So, you had 6 cases and assumed that each one would have a 1/6 chance to happen. The error is that the cases you list have not the same chance to occur, since the ones of 3 matches have half the chances of occuring than the ones of 2 matches.

Anyhow, if you're still unhappy with that, let me know and I'll give you a math demonstration of 1/4 = 1/3 = 1/2 = 2/3 (of course, it'll be cheaty, but it may be fun). :P
Now, no joking, I insist you on what I suggested on my previous post: once you have the first 2 results, they have to be enough to decide if a 3rd match is required. So, make the case breakdown for after the second match. You should see it clear then.
beta wrote:...and it seems like nobody remembered that there would be a third game or couldn't figure out how it would affect the answer.
Ok, you're right on the fact that there can be a third game, but due to the nature of the problem, we got the answer (there is a 3rd match or not) before the 3rd match, if it's finally done. This means that, since you by force know the answer before that 3rd match, the outcome of this one cannot affect that answer.
And this is my limit... I've teached maths to many people since 4 years ago or so (starting at secondary school, helping classmates, and afterwards helping other people for a nice price), but I don't feel able to explain you what's your mistake in many more ways. Try to think about that, and you should understand it.

Edit: PD: Wow! I just found another arguing for D! If you care the chance of the third game being cancelled, the chances of the event finishing after 2 matches go higher. ie: a game could be cancelled due to weather, violence on the playfield, too many harmed, sanctioned or ill players on both teams, and so on... the only limit is imagination XD Anyhow, both this arguing and the previous one 'bout this option are non-math, so they shouldn't be taken in account... I've put them just for fun :P

Edit: apologies for the "bump". I was just reviewing old posts, and I think I clicked the button by accident. :oops:


Last bumped by herenvardo on Sun Jan 27, 2008 11:56 pm.
I have failed to meet my deadlines so many times I'm not announcing my projects anymore. Whatever I'm working on, it'll be released when it is ready :P

Post Reply

Who is online

Users browsing this forum: No registered users